GRE Big Book 27 Analytical Puzzles Solution

You might also like

Download as pdf or txt
Download as pdf or txt
You are on page 1of 33

GRE Big Book

Test 27
Analytical
Section Solution
SECTION 3

Questions 1-7

At an art show, exactly five sculptures are to be displayed in a room with exactly five stands,
arranged along a single wall and numbered consecutively 1 through 5. The sculptures are to be
selected from a total of eight sculptures- M, N, P, Q, R, S, T, and U- and displayed, one sculpture
on each stand, according to the following conditions:

 Either M or U or both must be selected.


 If M is selected, M must be on stand 1.
 Either R or S must be on stand 3.
 If T is selected, P must also be selected, and T and P must then be on stands that are
immediately adjacent to one another.

Narrative Breakdown:
 Five sculptures are each to be in five stands numbered from 1 to 5.
 Sculptures selected would be from M, N, P, Q, R, S, T, and U.
 Either M/U/M+U must be selected.
 If M is selected, then it would be on stand 1.
 Either R/S must be on stand 3.
 If T is selected then P will also be selected. Both P and T will be on adjacent stands in the
form of T P or P T. Thus if P is not selected, T is not selected. But this does not mean that if
P is selected, T has to be selected.

1. Which of the following is an acceptable selection of sculptures to be displayed on stands 1


through 5?

1 2 3 4 5
A. M R T P Q
B. N T S U Q
C. P T S R U
D. T P R S M
E. U N Q P T

Solution:
If M is selected, it has to be in stand 1. M is selected but not in stand 1 in option D. So this
option can be eliminated.

Either R/S must be on stand 3 which they are not in options A and E. So both options can be
eliminated.
If T is selected, P must also be selected. In option B, T has been selected but not P. So this option
can be eliminated as well.

Answer is C

2. If S is on stand 1, which of the following must be true?


A. P is on stand 4.
B. R is on stand 3.
C. T is on stand 2.
D. T is on stand 4.
E. U is on stand 5.

Solution:
If S is on stand 1, R would be on stand 3. This is because either R/S has to be on stand 3.

Answer is B

3. If T is on stand 5, which of the following pairs of sculptures can be on stands that are
immediately adjacent to each other?
A. M and P.
B. Q and N.
C. Q and P.
D. R and T.
E. U and R.

Solution:
If T is on stand 5, P has to be selected and would be adjacent to T. So P would be in stand 4.
Either R/S would be on stand 3. The arrangement would be as follows:

1 2 3 4 5
R/S P T

M would not be adjacent to P, in the form of M P or P M, as stand 3 is filled by either R/S and
stand 5 is filled by T. Also Q would not be adjacent to P for the same reason. So both options A
and C can be eliminated.

R and T would not be adjacent to each other due to P being in the middle of R and T. So option
D can be eliminated.
Stand 1 and 2 can be filled with two sculptures. One of them needs to be either M/U as either
M/U/M+U must be selected. U can be on stand 2 and stand 1 could be filled be with either M, N
or Q. This can be shown as follows:

1 2 3 4 5
M/N/Q U R/S P T

U could be adjacent to R which is mentioned in option E. We do not have to concentrate on


option B anymore.

Answer is E

4. If U is on stand 4, any of the following can be on stand 5 EXCEPT


A. N
B. P
C. Q
D. R
E. T

Solution:
If U is on stand 4, the resultant arrangement would be as follows:

1 2 3 4 5
R/S U

T cannot be on stand 5 because then P would have to be on stand 4. P cannot be on stand 4 as U


is already on stand 4. Thus T is the only exception.

Answer is E

5. If T is on stand 2, which of the following sculptures must be selected?


A. M
B. N
C. R
D. S
E. U

Solution:
If T is on stand 2, P would be on stand 1 as stand 3 would be filled by either R/S. The resultant
arrangement would be as follows:
1 2 3 4 5
P T R/S

Either M/U/M+U must be selected. If M is selected it has to be on stand 1 which is not possible
in this arrangement as P is already on stand 1. So U will be selected to be on a stand instead of
M.

Answer is E

6. If P is not selected and R is on stand 1, which of the following lists, in alphabetical order,
those sculptures that must also be selected?
A. M, Q, T, and U
B. N, Q, S, and T
C. N, Q, S, and U
D. N, S, T, and U
E. Q, S, T, and U

Solution:
If P is not selected and R is on stand 1, S would be on stand 3 as stand 3 needs to be filled by
either R/S. If P is not selected, then T would not be selected as well. Thus we can eliminate
options A, B, D and E as all the arrangement in the options contain T.

Answer is C

7. If Q is displayed on a stand immediately adjacent to a stand on which R is displayed and


immediately adjacent to a stand on which S is displayed, which of the following must be
true?
A. N is on either stand 4 or stand 5.
B. Q is on either stand 2 or stand 4.
C. R is on either stand 1 or stand 3.
D. S is on either stand 3 or stand 5.
E. U is on either stand 2 or stand 4.

Solution:
If Q is adjacent of both R and S, this means that Q would be in the middle of R and S. This could
be either in the form of R Q S or S Q R. Also either R/S has to be on stand 3. The resultant
arrangement and its variations would be as follows:
A 1 2 3 4 5 C 1 2 3 4 5
R Q S R Q S

B 1 2 3 4 5 D 1 2 3 4 5
S Q R S Q R

From the variety of possible arrangements displayed above, it can be seen that Q can be on stand
2 or on stand 4 in all four occasions.

Answer is B
Questions 11-15

From time to time, the managing director of a company appoints planning committees, each
consisting of exactly three members. Eligible for appointment are three executives from Finance-
F, G, and H- and three executives from Operations- K, L, and M. Any given committee is subject
to the following restrictions on appointments:

 At least one member must be from Finance, and at least one member must be from
Operations.
 If F is appointed, G cannot be appointed.
 Neither H nor L can be appointed unless the other is appointed is also.
 If K is appointed, M must also be appointed.

Narrative Breakdown:
 Each planning committee will have 3 members.
 Finance- F, G, and H.
 Operations- K, L, and M.
 At least one from F, G, and H and at least one from K, L, and M must be appointed.
 If F is appointed, G cannot be appointed. So if G is appointed, F cannot be appointed.
 If H is appointed, then L is appointed. Similarly if L is appointed, then H is appointed.
 If K is appointed, M must also be appointed. So if M is not appointed, K cannot be
appointed. But this does not mean that if M is appointed, K has to be appointed.

11. Which of the following is an acceptable committee?


A. F, H, and M
B. G, L, and M
C. H, K, and L
D. H, L, and M
E. K, L, and M

Solution:
If H is appointed, then L is appointed. Options A, B and E can be eliminated as all three options
have either H/L missing while the other is present.

If K is appointed, M is appointed. Thus option C can be eliminated because M is not present in


the committee.

Answer is D
12. If appointees from Finance are in the majority on a committee, that committee must include
A. F
B. G
C. K
D. L
E. M

Solution:
If appointees from Finance are a majority on the committee, this would mean that 2 of the
members would be from Finance. If F is appointed, G is not appointed. If G is appointed, F is not
appointed. So only one of the two attendees can be chosen between F and G to be the first
member from Finance. The second member from Finance would then be H. If H is appointed
then L is appointed. Thus the committee must include L.

Answer is D

13. If appointees from Operations are in the majority on a committee, that committee must
include
A. F
B. G
C. K
D. L
E. M

Solution:
If appointees from Operations are a majority on the committee, this would mean that 2 of the
members would be from Operations. If K is appointed, M would be selected leaving us with
choosing either F/G for the committee. If M and L are appointed, then H would have to be
appointed as well. The two alternative committees can be illustrated below:

K M F/G or M L H

Both alternative committees have M common in them.

Answer is E
14. If F is appointed to the same committee as M, which of the following will be true of that
committee?
A. Appointees from Finance are in the majority.
B. Appointees from Operations are in the majority.
C. G is a committee member.
D. L is a committee member.
E. K is not a committee member.

Solution:
If F and M are appointed in the same committee, G cannot be appointed. The final slot would
have to be filled by either H/L/K.

F M L/H/K

If H is appointed then L would be appointed. If L is appointed then H would be appointed.

FMLH

This would make the number of members of the committee to be 4 which is not allowed. Thus
members of the committee will be F, M, and K. This makes appointees from Operations a
majority in the committee.

Answer is B

15. If the restrictions on appointments apply also to a four-member committee appointed from
the same group of executives, which of the following will be true?
A. If F is appointed, M must also be appointed.
B. If G is appointed, K must also be appointed.
C. If H is appointed, F must also be appointed.
D. If L is appointed, G must also be appointed.
E. If M is appointed, K must also be appointed.

Solution:
The committee would now consist of 4 members. In option A, if F is appointed, then G would
not be appointed. K cannot be appointed because if K is appointed then M would have to be
appointed which would lead to choosing between L and H for the final slot.

F K M L/H
The choice would not be possible to be made because if L is appointed then H would be
appointed or if H is appointed then L would be appointed.

FKMLH

This would make the number of members of the committee to be 5 which is not allowed. If we
choose F and M to be the members of the committee, then both L and H would be chosen to
make it a 4 member committee.

FMLH

Thus if F is appointed, M must also be appointed.

Answer is A
Questions 16-18

A psychologist has designed an experiment that involves running five mice- F, G, J, K, and M-
through a maze that is connected to five compartments- 1, 2, 3, 4, and 5. The psychologist places
each mouse in one of the five compartments. When a bell is rung, each mouse leaves its
compartment, runs through the maze, and enters or reenters one of the five compartments. At no
point is there more than one mouse in any compartment.

 When the bell is rung, any mouse placed is 4 always goes to 2, and any mouse placed in 2
always goes to 4.
 When the bell is rung, any mouse placed is 5 always goes to 3, and any mouse placed in 3
always goes to 5.

The psychologist has designed the experiment such that, after the mice have run through the
maze, the following outcomes always obtain:

 M is neither in 3 nor in 4.
 If J is in 1, K is in 2.
 If M is in 2, K is in 5.

Narrative Breakdown:
 Five mice (F, G, J, K, and M) run through five compartments numbered from 1 through 5.
 Each mouse may enter or reenter one of the compartments after the run has ended.
 After bell has rung:
 Mouse from compartment 4 would go to compartment 2.
 Mouse from compartment 2 would go to compartment 4.
 Mouse from compartment 5 would go to compartment 3.
 Mouse from compartment 3 would go to compartment 5.
 After the experiment is done:
 M will not be in either 3/4.
 If J is in 1, K is in 2. So if K is not in 2, J is not in 1. But if K is in 2, it will not
necessarily be that J is in 1.
 If M is in 2, K is in 5. So if K is not in 5, M is not in 2. But if K is in 5, it will not
necessarily mean that M is in 2.
16. Which of the following is a possible distribution of the mice after they have run through the
maze?
1 2 3 4 5
A. J K M F G
B. G M K J F
C. F J G M K
D. J M F K G
E. M K G F J

Solution:
M will not be in either 3/4 after the run. Options A and C can be eliminated as in both options, M
is either in 3/4.

If J is in 1 after the run, K will be in 2 after the run. K is not in 2 in option D, so option D can be
eliminated.

If M is in 2 after the run, K will be in 5 after the run. K is not in 5 in option B, so option B can be
eliminated too.

Answer is E

17. If M is in 2 after the mice have run through the maze, K must have been in which of the
following before running through the maze?
A. 1
B. 2
C. 3
D. 4
E. 5

Solution:
If M is in 2 after the run, K is in 5 after the run. As K is in 5 after the run, it must have been in 3
before the run. This is because all mice in 3 run to 5 after the bell rings.

Answer is C
18. If F is in 5 after the mice have run through the maze, M must have been in which of the
following before running through the maze?
A. 1
B. 2
C. 3
D. 4
E. 5

Solution:
If F is in 5 after the run, it must have been in 3 before the run. This is because all mice in 3 run to
5 after the bell rings. M cannot be in either 3/4 after the run. We can illustrate all these
information through a diagram below as follows:

1 2 3 4 5
Before the run F
After the run M M F

If F is in 3 before the run, M cannot be in 3 before the run.

1 2 3 4 5
Before the run FM
After the run M M F

As K is not in 5 after the run, M will not be in 2 after the run.

1 2 3 4 5
Before the run FM
After the run M M M FK

If M is not in 2 after the run, it is not in 4 before the run. This is because all mice in 4 run to 2
after the bell rings.

1 2 3 4 5
Before the run FM M
After the run M M M FK

If M is not in 3 after the run, it is not in 5 before the run. This is because all mice in 5 run to 3
after the bell rings.

1 2 3 4 5
Before the run FM M M
After the run M M M FK
If M is not in 4 after the run, it is not in 2 before the run. This is because all mice in 2 run to 4
after the bell rings.

1 2 3 4 5
Before the run M FM M M
After the run M M M FK

Thus the only compartment that M can be before the run is 1.

1 2 3 4 5
Before the run M M FM M M
After the run M M M FK

The answer is A
Questions 19-22

A detective watching suspects Q, R, S, and T and their movements in and out of a downtown
building has made the following observations:

 Every day, each of the suspects enters, and later leaves, the building exactly once.
 No suspect ever enters or leaves the building together with another suspect.
 No suspect ever leaves the building in the same position- first, second, third, or fourth- in
which he or she entered the building.
 Both in entering the building and in leaving it, R is always earlier than S.

Narrative Breakdown
 4 suspects - Q, R, S, and T.
 Each suspect enters and leaves the building once.
 No suspect enters or leaves the building with another suspect.
 No suspect leaves the building in the same position entered.
 R will always be earlier than S, while entering the building and also while leaving the
building.

19. Which of the following could be the order, from first to last, in which the suspects leave the
building on a day on which they enter it in the order T, Q, R, S?
A. Q, R, S, T
B. Q, R, T, S
C. R, Q, S, T
D. S, T, Q, R
E. T, R, S, Q

Solution:
If T enters the building 1 st, he or she will either be the 2nd, 3rd or 4th to leave the building. Option
E can be eliminated because in this option T is the 1st to leave the building.

If Q enters the building 2nd, he or she will either be the 1st, 3rd or 4th to leave the building. Option
C can be eliminated because in this option Q is the 2nd to leave the building.

If S enters the building 4th, he or she will either be the 1st, 2nd or 3rd to leave the building. Option
B can be eliminated because in this option S is the 4th to leave the building.

R will be earlier than S in leaving the building. In option D, S leaves in the building 1 st and R
leaves the building 4th. So option D can be eliminated.
Answer is A

20. If, on a certain day, Q and T enter the building second and third, respectively, and Q also
leaves the building before T does, the order in which the suspects leave the building, from
first to fourth, must be
A. Q, R, S, T
B. Q, R, T, S
C. Q, T, R, S
D. R, Q, S, T
E. R, S, Q, T

Solution:
If Q enters the building 2nd, he or she will either be the 1st, 3rd or 4th to leave the building. Option
D can be eliminated because in this option Q is the 2nd to leave the building.

If T enters the building 3rd, he or she will either be the 1st, 2nd or 4th to leave the building. Option
B can be eliminated because in this option T is the 3rd to leave the building.

If Q has entered the building 2nd and T has entered the building 3rd, the entrance schedule would
be as follows:
_ Q T_

As R has to enter the building earlier than S, and as the earliest position available is 1, R would
be the 1st to enter the building and S would the 4th to enter the building.

RQTS

If R enters the building 1st, he or she will either be the 2nd, 3rd or 4th to leave the building. Option
E can be eliminated because in this option R is the 1st to leave the building.

If S enters the building 4th, he or she will either be the 1st, 2nd or 3rd to leave the building. Option
C can be eliminated because in this option S is the 4th to leave the building.

Answer is A
21. On a day on which the two suspects who enter the building first are also the two suspects
who leave it first, the last two suspects to enter the building could be
A. Q and R
B. Q and T
C. R and S
D. R and T
E. S and T

Solution:
In option A, Q and R are the last to enter the building. The illustration would be as follows:

__QR

As it can be seen, S has to enter the building either 1st or 2nd, which is earlier than R. This is not
allowed because R will enter the building earlier than S does. So option A can be eliminated.

In option B, Q and T are the last to enter the building. R will then be the 1st to enter the building
and S will be 2nd to enter the building. This is because R will be entering the building earlier than
S does. The illustration would be as follows:

RSQT

According to the question, the first two suspects to enter the building will be the first two
suspects to leave the building. So R and S will then be the first two suspects to leave. But it is
also to be noted that the suspects have to leave in a different position than the one they entered
in. So R should be the 2nd to leave and S should be the 1st to leave. This is not possible as R has
to be always earlier than S in entering the building as well as leaving the building. So option B
can be eliminated.

In option C, R and S are the last to enter the building. Q will then enter the building in either the
1st/2nd position and then T will enter either the 2nd/1st position. This is because Q and T are
independent of any constraints. This can be illustrated as follows:

Q/T T/Q R S

As Q and T will be the first to leave, they can change their positions between them and still
fulfill the conditions. R would then have to be the 4th to leave the building. This is because it has
to leave the building in a different position than entered into. R cannot shift its position to 4th
while leaving the building because then S would be the 3rd to leave the building which is earlier
than R leaves the building. Option C can be eliminated.
In option D, R and T are the last to enter the building. This could be illustrated as follows:

__RT

As seen above, S would either be either 1st/2nd to enter the building which would be earlier than
R entering the building. Thus option D can be eliminated.

Answer is E

22. On a day on which R enters the building second and T enters it third, which of the following
must be true?
A. Q leaves the building first.
B. Q leaves the building third.
C. R leaves the building first.
D. S leaves the building third.
E. T leaves the building second.

Solution:
If R enters the building 2nd and T enters it 3rd, the illustration would be as follows:

_RT_

R would be entering the building earlier then S, so S would enter the building 4 th. This would
leave Q to enter the building 1st.

QRTS

Now for the leaving schedule, the alternative arrangements would be as follows:

R S Q T or R T S Q

In both arrangements R is the 1 st to leave the building.

Answer is C
SECTION 6

Questions 1-6

Seven airline flights- 101, 102, 103, 104, 105, 106, and 107- are to be scheduled for departure,
one at a time on the hour, from 9 a.m. until 3 p.m. The schedule must conform to the following
requirements:

 Flight 101 must depart at 9 a.m.


 Flight 105 must depart later than Flight 103, and also later than Flight 102.
 Flight 104, 106, and 107 must depart on consecutive hours in that order.

Narrative Breakdown:
 7 airlines (101, 102, 103, 104, 105, 106 and 107) travelling from 9 am till 3 pm.
 Each flight will depart at a specific time slot.
 101= 9 a.m.
 105 departs after both 102 and 103 have departed. It does not show, however, which flight
(102 or 103) will be departing before the other one (102 or 103) does. 102 could depart
before 103 departs. 102 could also depart after 103 departs.
 104 departs immediately before 106. 106 departs immediately before 107.

1. If Flight 107 is scheduled to depart at noon, Flight 105 must be scheduled to depart at
A. 10 a.m.
B. 11 a.m.
C. 1 p.m.
D. 2 p.m.
E. 3 p.m.

Solution:
We can structure the body of the puzzle as follows:

9 a.m. 10 a.m. 11 a.m. 12 p.m. 1 p.m. 2 p.m. 3 p.m.

101 would be departing at 9 a.m.

9 a.m. 10 a.m. 11 a.m. 12 p.m. 1 p.m. 2 p.m. 3 p.m.


101

107 departs at noon.

9 a.m. 10 a.m. 11 a.m. 12 p.m. 1 p.m. 2 p.m. 3 p.m.


101 107
106 would immediately depart before 107 departs and 104 would immediately depart before 106
departs.

9 a.m. 10 a.m. 11 a.m. 12 p.m. 1 p.m. 2 p.m. 3 p.m.


101 104 106 107

As 105 has to depart after both 102 and 103 has departed, and as there are only 3 time slots left,
105 has to depart in the last time slot available which is 3 p.m.

9 a.m. 10 a.m. 11 a.m. 12 p.m. 1 p.m. 2 p.m. 3 p.m.


101 104 106 107 102/103 103/102 105

Answer is E.

2. If Flights 103 and 104 are scheduled to depart at 11 a.m. and 12 noon, respectively, Flight
102 must be scheduled to depart at
A. 9 a.m.
B. 10 a.m.
C. 1 p.m.
D. 2 p.m.
E. 3 p.m.

Solution:
101, 103 and 104 would depart at 9 a.m., 11 a.m. and 12 noon respectively.

9 a.m. 10 a.m. 11 a.m. 12 p.m. 1 p.m. 2 p.m. 3 p.m.


101 103 104

106 will depart after 104 departs and 107 will depart after 106 departs.

9 a.m. 10 a.m. 11 a.m. 12 p.m. 1 p.m. 2 p.m. 3 p.m.


101 103 104 106 107

As 102 has to depart earlier than 105 departs, and as the earliest time slot available is 10 a.m.,
102 would be leaving at 10 a.m.

9 a.m. 10 a.m. 11 a.m. 12 p.m. 1 p.m. 2 p.m. 3 p.m.


101 102 103 104 106 107 105

Answer is B.
3. Which of the following lists three flights in a sequence, from first to last, in which they could
be scheduled to depart consecutively?
A. 101, 104, 103
B. 102, 103, 106
C. 104, 105, 106
D. 106, 107, 103
E. 106, 107, 104

Solution:
104, 106 and 107 have to depart consecutively which they do not is options A, B, C and E. So
these options can be eliminated.

Answer is D

4. If Flight 106 is scheduled to depart at 2 pm, Flight 105 must be scheduled to depart at
A. 10 a.m.
B. 11 a.m.
C. 12 noon
D. 1 p.m.
E. 2 p.m.

Solution:
101 and 106 would depart at 9 a.m. and 2 p.m. respectively.

9 a.m. 10 a.m. 11 a.m. 12 p.m. 1 p.m. 2 p.m. 3 p.m.


101 106

104 departs before 106 departs and 107 departs after 106 departs.

9 a.m. 10 a.m. 11 a.m. 12 p.m. 1 p.m. 2 p.m. 3 p.m.


101 104 106 107

102 and 103 have to both depart before 105 departs, so the latest time that 105 can depart is at 12
p.m.

9 a.m. 10 a.m. 11 a.m. 12 p.m. 1 p.m. 2 p.m. 3 p.m.


101 102/103 103/102 105 104 106 107

Answer is C
5. Which of the following must be true about the scheduled order of the flights?
A. Flight 103 is scheduled to depart later than Flight 102.
B. Flight 104 is scheduled to depart later than Flight 103.
C. Flight 105 is scheduled to depart later than Flight 104.
D. Flight 106 is scheduled to depart later than Flight 105.
E. Flight 107 is scheduled to depart later than Flight 106.

Solution:
Flight 107 has to depart after Flight 106 has departed.

Answer is E

6. What is the latest hour at which Flight 102 can be scheduled to depart?
A. 10 a.m.
B. 11 a.m.
C. 12 noon
D. 1 p.m.
E. 2 p.m.

Solution:
101 would depart at 9 a.m.

9 a.m. 10 a.m. 11 a.m. 12 p.m. 1 p.m. 2 p.m. 3 p.m.


101

As 104, 106 and 107 depart consecutively, they can all depart immediately after 101 has
departed.

9 a.m. 10 a.m. 11 a.m. 12 p.m. 1 p.m. 2 p.m. 3 p.m.


101 104 106 107

As 105 has to depart after 102 and 103 has departed, and as there are only three time slots
available, 105 can only depart at the last time slot which is 3 p.m.

9 a.m. 10 a.m. 11 a.m. 12 p.m. 1 p.m. 2 p.m. 3 p.m.


101 104 106 107 105

102 can depart before 103 departs. 102 can also depart after 103 departs.

9 a.m. 10 a.m. 11 a.m. 12 p.m. 1 p.m. 2 p.m. 3 p.m.


101 104 106 107 102/103 103/102 105
The latest time in which 102 can depart is 2 p.m.

9 a.m. 10 a.m. 11 a.m. 12 p.m. 1 p.m. 2 p.m. 3 p.m.


101 104 106 107 103 102 105

Answer is E
Questions 10-14

A farmer is deciding which crops to plant. Either three or four fields will be planted; in each field
only one crop will be planted. Exactly the same fields that are planted the first year will be
planted the second year, but no field will be planted to the same crop for two consecutive years.
For each field, the farmer will choose from among five possible crops- corn, soybeans, alfalfa,
rye and barley- according to the following conditions:

 In any year, at least one field will be planted to a cereal grain; the possible grains include
corn, rye, and barley only.
 The year after corn is planted in a field, either soybeans or alfalfa must be planted in that
field.

Narrative Breakdown:
 The crops can be labeled c, s, a, r, and b.
 Fields will be 3 or 4.
 Number of fields in first and second year will be same.
 No repetition of crops in two years in any field.
 Either c/r /b will be planted in minimum 1 field.
 If c is in a field in year 1, then either s/a must be planted in that field in year 2.

10. In a year in which corn is not planted, which of the following is true?
A. Either alfalfa or soybeans, but not both, must be planted.
B. Either barley or rye, or both, must be planted.
C. Both alfalfa and soybeans must be planted.
D. Both alfalfa and rye must be planted.
E. Either barley or soybeans must be the only crop planted.

Solution:
As either c/r /b has to be planted in minimum 1 field, if c is not planted then we have to fill the
fields with either r/b. r and b both can also be planted in the fields without restriction.

Answer is B
11. If the farmer plants three fields, each of the following is a possible selection of crops for the
first year EXCEPT
A. barley, barley, barley
B. barley, rye, soybeans
C. corn, corn, corn
D. corn, alfalfa, barley
E. rye, rye, rye

Solution:
If c is planted in all three fields in the first year, only s/a can be planted in each of the three fields
in the second year. But it is mentioned that either c/r/b must be planted in a minimum of 1 field
each year. So all three fields cannot be planted with just corn. Thus option C is the exception.

Answer is C

12. If the farmer plants three fields to corn, corn, and soybeans, respectively, which of the
following selections is possible for the same three fields the following year?
A. Alfalfa, alfalfa, barley respectively
B. Alfalfa, soybeans, soybeans respectively
C. Rye, rye, soybeans respectively
D. Soybeans, corn, corn respectively
E. Soybeans, rye, rye respectively

Solution:
If the first two fields are planted with c in the first year, then next year the same two fields will
be planted with either s/a. Option C can be eliminated as the two fields planted with r in the
second year needed to be planted with either s/a. Options D and E can be eliminated for similar
reasons.

Option B can be eliminated because if the third field to be planted in the first year is s, then in
the second year the same field needs to be planted with a different crop.

Answer is A
13. If the farmer plants four fields, with corn in two of the fields the first year, how many crops
must there be that are not planted the first year in any field but have to be planted the next
year in some field?
A. 0
B. 1
C. 2
D. 3
E. 4

Solution:
Four fields have been planted in the first year. Two have c in them. The rest are random. In year
2, four fields will also be planted. Either s/a will be in the fields in which c have been planted in
the first year. This can illustrated as follows:

1 2 3 4
Year 1 c c
Year 2 s/a s/a

We are to find out how many crops which are not planted in the first year in any of the four
fields will be planted in the next year in some field. If we plant either s/a in both field 3 and 4 in
the first year, we can plant c in both field 3 and field 4 in the second year without any problem.

1 2 3 4
Year 1 c c s/a s/a
Year 2 s/a s/a c c

A detailed illustration of different situations would be as follows:

1 2 3 4 OR 1 2 3 4
Year 1 c c s s Year 1 c c a a
Year 2 s s c c Year 2 a a c c

As we can see from above, all crops planted in the first year can be planted in the second year in
both situations. So there are no plants which are not planted in the first year but are planted in the
second year.

Answer is A
14. If the farmer plants four fields, the maximum number of fields that can be planted to grains in
both years is.
A. 0
B. 1
C. 2
D. 3
E. 4

Solution:
Four fields can be planted with r in the first year and either c/b in the second year.

1 2 3 4 OR 1 2 3 4
Year 1 r r r r Year 1 r r r r
Year 2 c c c c Year 2 b b b b

The fields can also be planted with b in the first year and r/c in the second year.

1 2 3 4 OR 1 2 3 4
Year 1 b b b b Year 1 b b b b
Year 2 r r r r Year 2 c c c c

The above illustrations all show that a maximum of four fields can be planted with grains in both
years. Chalking down any one of the illustrations above would have given us the answer.

Answer is E
Questions 15-17

A square, floating platform is supported at its corners by four hollow vessels that are labeled 1, 2,
3, and 4. 1 is diagonally across from 3, and 2 is diagonally across from 4. Three of the vessels are
each filled with a different liquid- N, O, or P- and the remaining vessel is empty, except while
there is a transfer of liquid in progress. The empty vessel can be filled to capacity by having all
of the liquid from one of the other three vessels pumped into it as follows.

 If 2 is empty, the liquid contained in 1 can be transferred to 2.


 If 3 is empty, the liquid contained in 2 can be transferred to 3.
 If 4 is empty, the liquid contained in 3 can be transferred to 4.
 If 1 is empty, the liquid contained in 3 or the liquid contained in 4 can be transferred to 1.
 No other transfers are possible.

O is the heaviest of the three liquids, and the platform is always tilted toward the vessel
containing O; the platform is tilted maximally when, and only when, the vessel containing O is
diagonally across from the empty vessel.

Narrative Breakdown:
 Four vessels 1, 2, 3 and 4.
 1 diagonally opposite to 3. 2 diagonally opposite to 4. This is the only information in regards
to formation that is important. Rest of the formation can change without any problem.
 Three liquids N, O and P for four vessels. One vessel remains empty all the time.
 Liquid can be transferred from one vessel to the other.
 L3/L4 to L1 to L2 to L3 to L4. (Liquid Transfer schedule)
 O heaviest and vessel containing O will tilt the most, especially when the diagonally opposite
vessel is empty.

15. If 1, 3, and 4 contain N, O, and P, respectively, and if there is then exactly one transfer of
liquid, that transfer must have which of the following results?
A. N is in 2.
B. O is in 1.
C. O is in 2.
D. O is in 4.
E. P is in 1
Solution:
Let us first put the information given in the question in a diagram.

1N 4P
2 3O
As 2 is empty, liquid from 1 has to be transferred to 2. So N will be in 2 after the transfer.

Answer is A

16. If P is in 4 and could be the next liquid to be transferred, which of the following must be
true?
A. The empty vessel is 1.
B. The empty vessel is 2.
C. The empty vessel is 3.
D. N cannot be the next liquid to be transferred.
E. O cannot be the next liquid to be transferred.

Solution:
If P is in vessel 4 and the next liquid to be transferred, it has to be transferred to an empty vessel
which would be 1.

1 4P
2 3

Answer is A

17. If 1, 2, and 4 contain O, N, and P, respectively, and if exactly three transfers are subsequently
made, which of the following could be the resulting contents of the four vessels?
1 2 3 4
A. Empty O P N
B. N P O Empty
C. O Empty N P
D. P N Empty O
E. P O N Empty

Solution:
The transfers can be shown as follows.

Original After First After Second After Third


Arrangement Transfer Transfer Transfer
1O 4P 1O 4P 1 4P 1P 4
2N 3 2 3N 2O 3N 2O 3N

The final transfer gives us the resulting contents of the four vessels.

Answer is E
Questions 18-22
Within an array there are four significant positions- positions 1 through 4. When the elements
forming the array- R, S, T, and U- are stationary, there is one of them in each of the four
positions. Periodically, there are reorderings of the elements in accordance with the following
laws:

 During each reordering, exactly one of the elements, the anchor, retains its position, and
each of the other three elements moves to a new position.
 Only R and U can be anchors.
 U can be the anchor only when it is in position 3 or position 4.
 Any reordering anchored by U must be followed by a reordering anchored by R.
 R can be the anchor in up to two consecutive reorderings.
 Any given array consists of the four elements in order of their positions from 1 through 4.

Narrative Breakdown:
 Positions will be from 1 to 4.
 R and U anchors.
 While each reordering, anchors remain in their position while the other elements switch their
places randomly.
 U can be anchor in 3rd or 4th position.
 If U is anchor in one reordering, R will be anchor in the next reordering.
 R can be anchor for two continuous reorderings only.

18. If U R T S is an array and if exactly one reordering occurs, which of the following can be the
array resulting from that reordering?
A. R S T U
B. S R T U
C. S R U T
D. T R U S
E. U S R T

Solution:
As U is not in either the 3rd/4th position in the original array, R is the anchor. R will remain in the
2nd position while the other elements change their position due to the reordering. So options A
and E can be eliminated as R changes its position in the reordering.

T should not remain in the 3rd position after the reordering which it does in option B. So option B
can be eliminated as well.
S needs to move away from the 4th position during the reordering which it does not in option D.
So option D can be eliminated.

Answer is C

19. If R T S U is an array and if exactly one reordering occurs, with the result that S is in position
1, which of the following must also be a result of that reordering?
A. R is in position 3.
B. R is in position 4.
C. T is in position 2.
D. T is in position 3.
E. U is in position 3.

Solution:
U is in position 4 which means U is the anchor. So if a reordering occurs which leads to S going
to position 1, the result will be as such:

S__U

The middle 2 positions will be filled with R and T by replacing their previous positions in the
reordering. T will shift from the 2nd position to the 3rd position and thus R will shift from the 1st
position to the 2nd position.

SRTU

Answer is D

20. If R moves from position 4 to position 1 in the course of a reordering, which of the
following can also occur in the course of that reordering?
A. S moves from position 1 to position 2.
B. S moves from position 1 to position 3.
C. S moves from position 1 to position 4.
D. T moves from position 2 to position 3.
E. T moves from position 3 to position 2.

Solution:
As R moves from the 4th position to the 1st position in the course of a reordering, U has to be the
anchor. U would then have to be in the 3rd position. The arrangement before the reordering
would be as follows:
__UR

The arrangement after the reordering would be as such:

R_U_

Option B can be eliminated because S cannot move from the 1st position to the 3rd position due to
the reordering as U is already in the 3rd position due to the reordering. Option D can be
eliminated for similar reasons.

T cannot move from the 3rd position to the 2nd position due to the reordering as T was not
originally in the 3rd position before the reordering, U was. So Option E can be eliminated too.

If S were to move from the 1st position to the 4th position, this would mean that T was in the 2nd
position before the reordering, and T is still in position 2 after the reordering. This is not possible
according to the conditions given which state that any element other than the anchor has to move
from its original position if reordering occurs. Option C can be eliminated as such.

Answer is A

21. If the array U S T R has resulted from a reordering, and if exactly two additional reorderings
occur, with the result that S is again in position 2, which of the following must have been the
array after the first of the two additional reorderings?
A. S T U R
B. T S U R
C. T U S R
D. U R S T
E. U T R S

Solution:
As U is not in the 3rd/4th position in the original reordering, R has to be the anchor for the next
reordering which is the first additional reordering. As R changes its position in the first
additional reordering in both Option D and E, both options can be eliminated.

As S is in the 2nd position in the original reordering, it cannot be in the 2nd position after the first
additional reordering. Thus Option B is eliminated.

As R has been the anchor twice, once in the original reordering and then in the first additional
reordering, U has to be the next anchor for the second additional reordering. U has to be in either
the 3rd/4th position to be the anchor which it is not in option C. So option C can be eliminated.
Answer is A

22. If S R U T is an array and if exactly two reorderings occur, the first anchored by R and the
second anchored by U, which of the following must be true directly after the second of the
two reorderings?
A. R is in position 1.
B. S is in position 1.
C. R is in position 2.
D. T is in position 3.
E. U is in position 4.

Solution:
After the first reordering, where R is the anchor, the arrangement will be as such:

_R__

If U has to be the anchor in the second reordering, it has to move to the 4 th position from the 3rd
position during the first reordering. This is because U can only be the anchor if it is in the 3 rd/4th
position. Thus as U was already in the 3rd position in the original arrangement, it can only go to
the 4th position during the first reordering to be the anchor for the second reordering. S will then
go to the 3rd position and T will go the 1st position during the first reordering. The resultant array
after the first reordering will be as such:

TRSU

After the first reordering is done and U has become the anchor, the second reordering will take
place. Only R, S and T will switch positions in the second reordering. The results of the second
reordering could be as follows:

R S T U or S T R U

One thing common between the two alternative arrangements is that although positions of R, S
and T can change in the arrangements, U stays fixed in the 4th position.

Answer is E

You might also like